« first day (572 days earlier)      last day (2046 days later) » 
05:00 - 15:0015:00 - 17:00

5:09 AM
@JohnRennie Are you there
 
@Abcd morning :-)
 
@JohnRennie morn, hi. Please see:
in Mathematics, 2 mins ago, by Abcd
Can someone please explain point (ii)
in Mathematics, 2 mins ago, by Abcd
It says: if A is singular matrix and (adj A) D = O then the system of equations given by AX = D is consistent with infinitely many solutions. Why is that?
 
Linear algebra?
 
Its for finding solution of:
$a_1 x + b_1y + c_1 x = d_1$, $a_2x....$ and till $a_3 x....$
@JohnRennie Just multiply LHS and you will get this system from definition of equality of matrices^
 
Aha, yes, the technique is often introduced as a way of finding solutions to simultaneous equations. Typically students aren't told that this is their first exposure to the subject of linear algebra.
 
5:14 AM
@JohnRennie Oh okay. I dont know what linear algebra is.
 
@Abcd it doesn't matter anyhow ...
If you have a set of simultaneous equations then they may have one solution, no solution or infinitely many solutions.
 
@JohnRennie Okay, can you explain point 2?
@JohnRennie yes
 
The solution to simultaneous equations is actually a matrix operation.
 
ikr
 
The conditions ii and iii are conditions on the matrices involved in the solution that tell whether the equations have one, none, or infinitely many solutions.
 
5:17 AM
ikr
 
Are you asking for a proof of point ii?
 
@JohnRennie Yes, I want to know how he got that.
 
Offhand I don't know the proof. It's the sort of thing you learn for exams then forget because you never need it again.
 
@JohnRennie i cant find it anywhere online too.
 
Do you need to know the proof? Obviously if $\det(A)=0$ everything blows up so there's no single solution.
 
5:27 AM
@JohnRennie there some significance of adjoint here
 
@Abcd not really. It's just that $A^{-1} = adj(A)/|A|$
 
@JohnRennie Does $\text{adj(A)D}= O $ make any physical sense to you?? (its given in point 2)
 
$\text{adj(A)D}= 0$ makes the right hand side $0/0$ and that is indeterminate, hence there are infinitely many solutions. Physically it means that two (or more) of your simultaneous equations aren't independent.
 
@JohnRennie which RHS?
 
$ X = A^{-1}D$
The authour splits $A^{-1}$ into $A^{-1} = adj(A)/|A|$
$$ X = \frac{adj(A)D}{|A|} $$
 
5:33 AM
Sorry wasnt paying attention to det(A)=0
@JohnRennie First of all inverse doesn't exist so its illegal to multiply.
$AX = D$ should be considered.
 
The adjoint always exists whether A is singular or not.
So you can always multiply $adj(A)$ and $D$
The only problem comes when you divide by the determinant.
The point is that if the determinant is zero you get problems, but you have to distinguish between $0/0$ and $x/0$ (for some non-zero $x$)
$0/0$ is indeterminate while $x/0$ is undefined.
 
Wait please.
AX = D.
How does adoint come in the story?
 
The inverse is the adjoint divided by the determinant
 
3 mins ago, by Abcd
@JohnRennie First of all inverse doesn't exist so its illegal to multiply.
@JohnRennie We are talking about singular matrix
inverse doesnt exist
 
OK we have a matter of definitions here I think
The inverse is always $adj(A)/|A|$
When we say the inverse doesn't exist that means $|A|=0$ and of course you cannot divide by zero
But there's nothing wrong with splitting the inverse into the adjoint and determinant and postponing the question of whether $|A|=0$ or not until later.
 
5:43 AM
@JohnRennie Okay but in point (ii) we have $X = \text{null matrix}/0$ not $0/0$
 
In (ii) you get a matrix whose elements are all equal to $0/0$ i.e. a matrix all of whose elements are undetermined
 
Yeah got it.
Got it, thanks!
Basically in 3 all solutions tend to infinity?
implies no solution.
 
Yes
 
@JohnRennie is this faster then Cramer's rule?
 
@Abcd it is effectively what you're doing when you use Cramer's rule.
In practice for small equations (2 or 3 variables) I'd probably just solve them by hand, but in physics we often get massive sets of equations and have to solve them by computer. That algorithm will use the approach decribed in your book not Cramer's rule.
 
6:35 AM
@sammygerbil I thought so.
 
6:55 AM
@JohnRennie. There?
I wanted to know what amount of weight would act during extension of a rod hanging freely from a ceiling due to its own weight?
 
@NehalSamee morning :-)
 
@JohnRennie morning.. :)
 
The stretch isn't uniform i.e. the bits of the rod near the ceiling are stretched more than the bits of the rod near the lower end.
This makes sense because the bits of the rod near the ceiling have almost the whole mass of the rod below them and stretching them.
 
@JohnRennie Why is that so? I only know that tension is more at the upper end
 
While the bits of the rod near the end have almost no mass below them pulling them down.
 
6:58 AM
@JohnRennie OK
 
What is it you want to know? How to calculate the extension of the rod?
 
So.. What amount of original weight would act?
@JohnRennie I heard its 1/3 ..how?
 
@NehalSamee have a look at Sammy's answer here:
3
Q: Why does different part of a spring having mass expand proportional to their distance while the spring has some mass hanged in the bottom?

Nobody recognizeableWhy do different parts of a massive spring expand proportional to their distance while the spring has some mass hung on the bottom which is comparably very less than the mass of the spring? If we take a massless (or practically negligible mass) spring then the elongation of the spring is uniform....

 
@JohnRennie this case is dynamic extension as Sammy said, but what's static extension... Is it the horizontal one?
 
I'm not sure why Sammy uses the terms dynamic extension and static extension.
It's just the difference between the spring being stretched by its own weight, i.e. hanging vertically, and not being stretched by its own weight, i.e. resting on a horizontal surface.
 
7:13 AM
@JohnRennie but masses considered are different... M/3 and M/2 ...
 
Ah, OK, actually Sammy's answer is more general that I realised at a first glance.
If you have a massive spring hanging vertically there are two situations to consider.
If the spring of mass $m$ and force constant $k$ is just hanging stationary then its extension is given by $mg/(2k)$ i.e. the extension is half of what you'd get if you hung a mass $m$ on the end of a weightless spring.
But suppose you set the spring oscillating. In that case the mass of the spring has two effects. Firstly it creates the downward force on the spring due to gravity, and secondly it affects the acceleration of the various parts of the spring due to Newton's second law.
So the oscillating case is more complicated than the static case.
 
7:47 AM
@JohnRennie Are you there
 
@Abcd hi
 
@JohnRennie reflection matrix for reflection about y =x tan theta
how to find it
 
As I recall when you're trying to construct a transformation matrix you do it by considering what happens to the unit vectors (1,0) and (0,1)
The vector (1,0) gets rotated by an angle $2\theta$ so it would go to $(\cos 2\theta, \sin 2\theta)$
and the vector (1,0) gets rotated clockwise (negative angle) by $90-\theta$, so it goes to, erm, I need to draw this ...
 
@JohnRennie Do you mean $\hat i$ and $\hat j$ respectively
 
Yes
Just draw the diagram and it's obvious ...
I can draw you a diagram to show what I mean if you want ...
 
7:55 AM
yes
 
OK, give me a moment ...
 
@JohnRennie I dont get this first.
 
The dashed line is the line $y = x\tan\theta$
So reflection in that line rotates (1,0) by an angle $2\theta$
 
1 min ago, by Abcd
@JohnRennie I dont get this first.
@JohnRennie Please see this . I dont get why we have to consider 1,0 and 0,1
 
Suppose you have some transformation matrix $( (a,b), (c,d) )$
When you multiply it by $(1,0)$ the result is $(a,b)$
So the top row of the transformation matrix is the same as the point that $(1,0)$ maps to.
 
8:05 AM
@JohnRennie no its a,c
If a b is first row. And c d is second row
 
Hmm, yes, maybe I mixed up my rows and columns.
But the basic principle remains true. By considering what the transformation does to (1,0) and (0,1) you can construct the transformation matrix.
Hmm, yes, it is the columns not the rows. Blame aging memory.
$$ \left( \begin{matrix} a & b \\ c & d \end{matrix} \right) \left( \begin{matrix} 1 \\ 0 \end{matrix} \right) = \left( \begin{matrix} a \\ c \end{matrix} \right) $$
$$ \left( \begin{matrix} a & b \\ c & d \end{matrix} \right) \left( \begin{matrix} 0 \\ 1 \end{matrix} \right) = \left( \begin{matrix} b \\ d \end{matrix} \right) $$
 
@JohnRennie So what if we are getting a c and b d back?
 
If you look at my diagram, $(1,0)$ transforms to $(\cos2\theta, \sin2\theta)$
So we immediately know that the transformation matrix for reflection in $y=x\tan\theta$ has $a = \cos2\theta$ and $c=\sin2\theta$.
 
3 mins ago, by Abcd
@JohnRennie So what if we are getting a c and b d back?
 
@Abcd I don't understand what you are asking
 
8:15 AM
Suppose we have a point x and y.
And we reflect it in y = x tan theta
So you are saying consider the unit vectors of x and y?
Then transform them?
Then multiply To get new position?
 
No. All we are asked to do is find the transformation matrix. The matrix doesn't depend on the values of $x$ and $y$ (obviously) so if we can find the matrix for some specific choice of $x$ and $y$ we have found the matrix.
 
just a min
Let me show from my textbook
 
What I'm saying is that if we consider the specific points (1,0) and (0,1) we can easily find the matrix
 
That's what I'm trying to explain, but you don't seem to want to follow my explanation.
I have no idea what the authors procedure is. He doesn't give any explanation for how he derived his result.
 
8:23 AM
Yes... that's what confused me there
 
What I'm explaining is a guaranteed way to get the trasnformation matrix.
 
Okay, I will ignore this idiot author.
@JohnRennie please explain the use of this part^
 
@Abcd which part?
 
@JohnRennie the message I have replied to. What is the use of doing that?
I mean we are getting a and c back in a new matrix, so how does that help?
 
We have some transformation matrix $((a,b),(c,d))$ where all of $a$, $b$, $c$ and $d$ are unknowns. Yes?
 
8:27 AM
What is a transformation matrix really?
(no definition in my book)
 
Suppose you have some transformation, e.g. reflection or rotation, that takes the point $(x,y)$ to $(X,Y)$. Then we can write the transformation as the matrix equation:
$$ Ax = X $$
Where $x = (x,y)$ and $X=(X,Y)$ (sorry slightly rubbish choice of notation).
Or if we write this out in full:
$$ \left( \begin{matrix} a & b \\ c & d \end{matrix} \right) \left( \begin{matrix} x \\ y \end{matrix} \right) = \left( \begin{matrix} X \\ Y \end{matrix} \right) $$
The matrix $A$ is the matrix that describes the transformation, and that's what I am calling the transformation matrix.
 
@JohnRennie I see, then?
@JohnRennie yes
 
And our task is to find the numbers $a$, $b$, $c$ and $d$.
 
yes
 
Now suppose we work out where the transformtion takes the point $(1,0)$. We work this out by whatever construction is appropriate. This is usually easy because it's usually easy to work out where $(1,0)$ goes. OK so far?
 
8:33 AM
yes
 
Suppose we find that $(1,0)$ maps to $(X,Y)$. Then we know that:
$$ \left( \begin{matrix} a & b \\ c & d \end{matrix} \right) \left( \begin{matrix} 1 \\ 0 \end{matrix} \right) = \left( \begin{matrix} X \\ Y \end{matrix} \right) $$
 
ya
 
If we hand multiply the matrices we get $X=a$ and $Y=c$
 
yes
 
So if we know $X$ and $Y$ that means we have found the first column of our transformation matrix.
 
8:35 AM
Ya
 
35 mins ago, by John Rennie
user image
 
ya
X = cos 2theta
Y = sin 2 theta
So we get a and c.
Then?
 
Yes
OK now we do the same thing for the point $(0,1)$. We figure out where this point goes under the transformation. I can give you the answer or you can have a go at working it out ...
 
jam
@JohnRennie I am getting sin 2 theta , -cos 2 theta
 
@Abcd yes, so now you know the second column of your transformation matrix.
i.e. you now know the values of $b$ and $d$
 
8:46 AM
@JohnRennie Got it thanks!
@JohnRennie For how much time are you here
 
This is a general technique that works for all linear transformations
@Abcd I'm around for at least a couple of hours
 
@JohnRennie ya its brilliant. Nothing like this is given in my book
 
@Abcd I'm a bit surprised because it is so simple to use
 
@JohnRennie But there's an assumption. How can we assume that every transformation can be represented by those matrices?
 
Offhand I can't remember the proof, but I don't think it's hard. The key assumption is that the transformation is linear. I'd have to Google to find the details.
In mathematics, a linear map (also called a linear mapping, linear transformation or, in some contexts, linear function) is a mapping V → W between two modules (including vector spaces) that preserves (in the sense defined below) the operations of addition and scalar multiplication. An important special case is when V = W, in which case the map is called a linear operator, or an endomorphism of V. Sometimes the term linear function has the same meaning as linear map, while in analytic geometry it does not. A linear map always maps linear subspaces onto linear subspaces (possibly of a lowe...
See the Matrices section of that article
 
8:53 AM
@JohnRennie thats not high school level stuff
@JohnRennie Can you tell me the simple reason?
 
@Abcd agreed, that's taking you down the rabbit hole of linear algebra. It's a fascinating subject but you don't need it for JEE you just need to know the main results.
@Abcd I don't think there is a simpler explanation than the one in the Linear map article. For now just accept that it's true and move on.
 
9:24 AM
@JohnRennie good morning
 
@harambe morning :-)
 
In water would there will be additional boyant force that we have to consider here?
But how will it help in the FBD... I mean it is not influencing the angle or the electric charge right?
 
Yes. The increased dielectric constant reduces the electrostatic force so the spheres would tend to hang more nearly straight down. However at the same time the buoyant force decreases the downwards force on the spheres and that compensates for the decrease in the electrostatic force.
 
Okay. Got it
 
Suppose we write the electrostatic force as $F_e$ and the net vertical force ($mg$ minus the buoyant force) as $F_v$, than the tan of the angle is $F_e/F_v$.
 
9:31 AM
Yeah
 
And you're told that this stays constant, so the fractional decrease in $F_e$ must be equal to the fractional decrease in $F_v$.
 
Okay
@JohnRennie About the angle.... I think I am confused. I thought the angle between the horizobtal and vertical force is 90..the electric force horizobtally and the Fv vertical ly
Never mind. I got it
 
Cool :-)
 
9:47 AM
@JohnRennie got the answer. Your method looked way clean than the solution showed
 
@harambe naturally :-)
 
XD
@JohnRennie if two spheres having say charges Q1 and Q2 are touched then the resultant charge on them will be Q1+Q2/2 on each
Would this mean electric force will change too
 
Well the force is initially $kQ_1Q_2/r^2$ and after they have touched it is $k(Q_1+Q_2)^2/(4r^2)$
If you set the two forces equal you'll find it is only true for specific values of $Q_1$ and $Q_2$
 
That's why... I had this question in a test and I just wrote it will be greater than the initial force but it was said it could be equal too....
 
10:12 AM
My expression for restoring force is: -kq^2y/2(a^2+y^2)^3/2
How do I proceed from here
I have ignored mg because its a constant force here
 
Use a binomial expansion.
 
I think there is some binomial series after this but how to use it
Should I only include the first two terms of the BT?
Will others be negligible
 
The $y \ll a$ is a dead giveaway. Expand the force as a power series in $y/a$ and discard terms in $(y/a)^2$ and higher.
 
Got it
 
Is there a mistake in that question? Should it be two charges at $x =+a$ and $x=-a$. That would make more sense
 
10:22 AM
@JohnRennie yes. I check it with the original question of jee and this mistake is rectified there so thee is a misprint here
 
Thought so :-)
Ah, no.
 
I am getting $y^3$
 
It's (a)
 
It doesn't match the options.....
@JohnRennie yes
 
It can't be c or d because the force is zero when $y=0$
And if you move in the positive $y$ direction the force is positive, so it has to be $+y$ not $-y$.
I can have a go at doing the calculation in bit, but right now I'm tied up in an online chat thing.
 
10:28 AM
Okay
I can do other questions meanwhile
 
11:07 AM
@harambe actually I don't think you need a binomial expansion. It's simpler than that.
 
Okay. Can you tell me about it
 
Let me draw a diagram ...
$$ F =\frac{k Q/2 Q}{r^2} $$
The vertical component is $F\sin\theta = F (y/r)$. And double that because we have a force from both sides. So we end up with:
$$ F_v =\frac{k Q^2 y}{r^3} $$
@harambe OK so far?
 
Yeah
 
And $r^2 = a^2 + y^2$ so we get:
$$ F_v =\frac{k Q^2 y}{(a^2 + y^2)^{3/2}} $$
 
Okay
 
11:19 AM
And if $y \ll a$ then $a^2 + y^2 \approx a^2$ so we just get:
$$ F_v \approx \frac{k Q^2 y}{a^3} $$
Or $F_v \propto y$
 
Got it. So we neglect y/ a fraction
@JohnRennie when we keep two spheres in contact then I read that charges will redistribute.... Why does this happen
 
It assumes the spheres are conductors. They don't have to be good conductors, just as long as they aren't perfect insulators.
When you touch two conductors together you effectively have a single conductor. And charge always distributes itself over the surface of a conductor.
If the balls are the same size then the symmetry means you'll end up with the same charge on each ball.
 
Okay
Can you help me with a similiar question based on this
 
Question 9?
 
11:35 AM
Yeah. About the spheres one l
 
OK, when you connect the two spheres their voltage has to be the same. Yes?
 
eh Voltage .... How is it a factor here
 
@harambe Because the charge on an object is related to the voltage and capacitance of the object by the usual $Q = CV$.
Hmm, actually we could do the problem without needing to invoke capacitance. Maybe that would be more intuitive.
 
Okay. It will be better if I could understand the method intuitively so that I can remember it easily
 
Suppose the charge on the first sphere is $Q_1$. Then the electrostatic potential at the surface of the sphere is $V = kQ_1/R$. OK so far?
 
11:42 AM
Ok
 
And likewise for the second sphere we get $V = kQ_2/(3R)$
 
Okay
 
The thing is that we've connected the two spheres with a conducting wire, so electrons can freely move along the wire from one sphere to the other. At equilibrium the potential has to be the same at both ends of the wire otherwise charge would keep flowing along the wire until the potentials became the same.
And that means $V_1 = V_2$
 
Okay
 
So that means:
$$ \frac{kQ_1}{R} = \frac{kQ_2}{3R} $$
$$ Q_2 = 3Q_1 $$
 
11:47 AM
Okay and now we can calculate the electric field of the spheres
One doubt
 
@harambe Yes?
 
When you said potential then about which point were you talking potential about
On the sphere
 
If you're outside a spherically symmetric charge then it behaves like a point charge at the centre of the sphere. Yes?
 
Yea.
For points outside the sphere
 
So at a distance $r \ge R$ from the first sphere the potential energy is just the usual $kQ/r$.
 
11:51 AM
Oh so you were talking about the portion of wire attached to the spherical surface?
 
The wire is connected to the surfaces of the spheres
 
Got it
 
12:05 PM
@JohnRennie that means... If the spring is static in vertical position without mass, then half of weight acts for extension... If it's oscillating with/without mass, then one third of weight acts... Right?
 
 
2 hours later…
1:51 PM
@JohnRennie are you interested in about maximum / minimum of functions
 
@harambe do you mean can I asnwer questions on it? If so, yes, I'm willing to give it a go.
 
Awesome. Yes
I need to find the range if this function
So I have been taught that I have to differentiate this function, find the critical points. That will give me local macima/ minima
 
OK, so if there is no maximum or minimum in the domain $x=0$ to $3$ then then the range is just from $f(0)$ to $f(3)$. Yes?
 
why?
 
There are only three cases:
1. the function changes smoothly from $x=0 \to 3$. In that case the maximum and minimum values are at the ends of the domain i.e. at $x=0$ and $x-=3$. Yes
 
2:02 PM
What do you mean by " changes smoothly "
 
Let me draw a diagram ...
I've drawn two functions. The red curve increases monotonically in the range 0-3 and green decreases monotonically in the range.
In both cases the maximium and minimum values are at the ends of the range 0-3.
 
Okay
 
But case 2 is when the function has a maximum:
In that case the minimum value is at one of the ends of 0-3, but the maximum is not in between at some value we have to find.
And I won't draw it, but case 3 is if the function has a minimum in 0-3
So we need to find out if our function has any maxima or minima in the range $x = 0 \to 3$
 
So In case 2 we know the minimum is at one of the end points but not the maximum... It can be anywhere between them
 
Correct. So we need to evaluate $f(x)$ at both the end points and at the maximum
 
2:12 PM
Okay
 
Do you know how to find the maxima and minima of a function?
 
Yea
 
So go ahead. Find the values of $x$ for the maxima and minima.
 
At maxima and minima the values are -14 and 13
At end values of function at 0 and 3 the values are 6 and -3
So range comes out to be (-14, 3)
Does this look right
 
Aren't you only considering the interval 0 to 3?
 
2:21 PM
Oops... (-14, 13)
 
That's your function in the interval 0 to 3
 
Okay.... Let me try again. Btw what is the website for this graph plotter
 
I used Microsoft Excel
 
The range looks like [-14, 6]
 
Yes
 
2:26 PM
I need to get that too.
I got -14
 
You can use Google Sheets. For simple stuff that is as good as Excel.
@harambe The function has a maximum at $x=-1$ and a minimum at $x=2$. Yes?
 
Yea
I am getting difficult y in maximum part
 
So we need to evaluate $f(0)$, $f(3)$ and $f(2)$ to be sure of covering the whole range.
 
Yea
 
We don't care about $x=-1$ because it's outside our interval of 0-3
 
2:30 PM
@harambe are preparing for jee?
 
Oh yea.. That why I was getting the wrong answer
@Fawad yes unfortunately
 
You are from cbse?
 
Yes
@JohnRennie is the steps combination of case 1 and case 2 or just case 2
I mean the steps of solution here
 
This is case 3 i.e. it has a minimum within the interval we are looking at.
 
So we do the solution and look for the case the solution falls into?
 
2:35 PM
First find the maxima and minima.
 
@JohnRennie how do you prove that image of orthocentre for a triangle lies on cirmumcentre of that triangle with respect to any side?
 
Then if the $x$ position of the maxima and minima lie outside the interval there is nothing more to do. Just evaluate $f(x) at the end points of the interval.
@Fawad no idea, I would have to Google it.
@harambe does that make sense so far?
 
Yea
 
2
Q: Why would the reflections of the orthocentre lie on the circumcircle?

maths loverLet ABC be a triangle which it not right-angled. Define a sequence of triangles $A_iB_iC_i$,with $i \geq 0$, as follows: $A_0B_0C_0$ is the triangle $ABC$; and, for $i \geq 0$, $A_{i+1}$, $B_{i+1}$, and $C_{i+1}$ are the reflections of the orthocentre of triangle $A_iB_iC_i$ in the sides $B_iC_...

 
@harambe If there are any maxima or minima in the interval then we need to find them and evaluate $f(x)$ at all the maxima and minima as well.
 
2:38 PM
Okay
 
In this case there was just a minimum in the interval.
 
Yea
 
But if the interval had been -2 to +3 then there would have been both a minimum and a maximum so we'd need $f(x)$ at both those values as well as at the ends of the interval.
 
Got it but if the question had one of the end points in open bracket ie they were not in domain then how can we proceed there?
 
@harambe not sure to be honest. Suppose in this case 0 and 3 weren't in the domain. In that case I guess you'd have write the upper limit of the range a $6 - \epsilon$, where $\epsilon$ is arbitrarily small.
To be honest my grasp of the more formal end of maths isn't good enough for me to know what the answer is.
 
2:48 PM
Oh kay. It's no problem. I will ask this on the math chat
 
@JohnRennie if domain is [0,3) then range is written as [-14,6) (if f(3)=6)
 
@Fawad Yes, that makes sense. @harambe
 
@JohnRennie but max is not given by extreme hmmm
 
I think I get it. If the function is continious the finding the limit at both points will give the value of function there maybe?
Yea. Continious function can flactuate but momotonous function can't. But still finding limit wont hurt right
 
I knew it was dumb comment so I deleted it
 
2:55 PM
@harambe You always need the values of $f(x)$ at the ends of the interval. Just because there is a maximum and/or minimum in the interval doesn't mean those are the highest/lowest values of the function in the interval.
Suppose in your question you had been given the interval -100 to +100. In that case the range is from $f(-100)$ to $f(100)$ because those are lower and higher than the values at the minimum and maximum.
 
Why does this happen only at end points
Why do we check only for end points
Because they are influencing how the graphs can be made?
Cases to be exact
 
If there is no maximum or minimum the function changes monotonically between the end point. Yes?
 
Yes
 
If there is a single maximum the function changes monotonically from the start point to the maximum, then monotonically from the maximum to the end point. Yes?
 
05:00 - 15:0015:00 - 17:00

« first day (572 days earlier)      last day (2046 days later) »